Help Finding Limit: Homework Statement

  • Thread starter Thread starter fishingspree2
  • Start date Start date
  • Tags Tags
    Limit
Click For Summary
The limit being discussed is \(\lim_{x \to 0} (1 - \cos x)\), which approaches 0 as \(x\) approaches 0. However, when considering \(\lim_{x \to 0} \ln(1 - \cos x)\), the argument of the logarithm approaches 0, leading to negative infinity. The discussion emphasizes the importance of evaluating limits from both the left and right to determine if they agree for the limit to exist. It suggests that using L'Hôpital's rule could provide an analytical solution to the limit. Understanding these concepts is crucial for mastering calculus fundamentals.
fishingspree2
Messages
138
Reaction score
0

Homework Statement


\mathop {\lim }\limits_{x \to 0} (1 - \cos x)

2. The attempt at a solution

well as x goes to 0, cos x goes to 1... ln(1-1) is undefined
now if I forget about plugging in x=0 and think a little bit, the ln argument gets very very small, and the logarithm of a decimal number is a negative number... so I would say the limit is minus infinity.

however, is it possible to get that result analytically... by transforming/simplyfing/etc. the function?

sorry, I have just finished precalculus and I am beginning calculus, so my calculus skills are crap =\

thank you
 
Last edited:
Physics news on Phys.org
hmm ... problem is: \lim_{x\rightarrow0}(1-\cos x)?

Can't you just plug it in?

Or is it:

\lim_{x\rightarrow0}\ln{(1-\cos x)}

Ok so have the graph infront of you. You know as you "approach" coming from the right, it's negative infinity ...

\lim_{x\rightarrow0^{+}}\ln{(1-\cos x)}=-\infty

What about from the left? Because in order for the Limit to exist, the L & R Limits must agree. But for the natural logarithm, how is it uniquely defined?
 
Last edited:
Once you have learned L^Hopitals rule, you will be able to apply it to this equation and get the same answer analytically.
 
Question: A clock's minute hand has length 4 and its hour hand has length 3. What is the distance between the tips at the moment when it is increasing most rapidly?(Putnam Exam Question) Answer: Making assumption that both the hands moves at constant angular velocities, the answer is ## \sqrt{7} .## But don't you think this assumption is somewhat doubtful and wrong?

Similar threads

Replies
6
Views
2K
Replies
32
Views
3K
  • · Replies 1 ·
Replies
1
Views
2K
  • · Replies 19 ·
Replies
19
Views
2K
Replies
35
Views
4K
  • · Replies 15 ·
Replies
15
Views
2K
  • · Replies 7 ·
Replies
7
Views
2K
  • · Replies 8 ·
Replies
8
Views
2K
Replies
3
Views
1K
  • · Replies 3 ·
Replies
3
Views
1K